Đến nội dung

Hình ảnh

China Western Mathematical Olympiad 2011


  • Please log in to reply
Chủ đề này có 2 trả lời

#1
Zaraki

Zaraki

    PQT

  • Phó Quản lý Toán Cao cấp
  • 4273 Bài viết

Day 1


$\fbox{1}.$ Given that $0 < x,y < 1$, find the maximum value of $\frac{xy(1-x-y)}{(x+y)(1-x)(1-y)}$

$\fbox{2}.$ Let $M$ be a subset of $\{1,2,3... 2011\}$ satisfying the following condition:
For any three elements in $M$, there exist two of them $a$ and $b$ such that $a|b$ or $b|a$.
Determine the maximum value of $|M|$ where $|M|$ denotes the number of elements in $M$

$\fbox{3}.$ Let $n \geq 2$ be a given integer
$a)$ Prove that one can arrange all the subsets of the set $\{1,2... ,n\}$ as a sequence of subsets $A_{1}, A_{2},\cdots , A_{2^{n}}$, such that $|A_{i+1}| = |A_{i}| + 1$ or $|A_{i}| - 1$ where $i = 1,2,3,\cdots , 2^{n}$ and $A_{2^{n} + 1} = A_{1}$
$b)$ Determine all possible values of the sum $\sum \limits_{i = 1}^{2^n} (-1)^{i}S(A_{i})$ where $S(A_{i})$ denotes the sum of all elements in $A_{i}$ and $S(\emptyset) = 0$, for any subset sequence $A_{1},A_{2},\cdots ,A_{2^n}$ satisfying the condition in $a)$

$\fbox{4}.$ In a circle $\Gamma_{1}$, centered at $O$, $AB$ and $CD$ are two unequal in length chords intersecting at $E$ inside $\Gamma_{1}$. A circle $\Gamma_{2}$, centered at $I$ is tangent to $\Gamma_{1}$ internally at $F$, and also tangent to $AB$ at $G$ and $CD$ at $H$. A line $l$ through $O$ intersects $AB$ and $CD$ at $P$ and $Q$ respectively such that $EP = EQ$. The line $EF$ intersects $l$ at $M$. Prove that the line through $M$ parallel to $AB$ is tangent to $\Gamma_{1}$


Day 2


$\fbox{1}.$ Does there exist any odd integer $n \geq 3$ and $n$ distinct prime numbers $p_1 , p_2, \cdots p_n$ such that all $p_i + p_{i+1} (i = 1,2,\cdots , n$ and $p_{n+1} = p_{1})$ are perfect squares?

$\fbox{2}.$ Let $a,b,c > 0$, prove that
\[\frac{(a-b)^2}{(c+a)(c+b)} + \frac{(b-c)^2}{(a+b)(a+c)} + \frac{(c-a)^2}{(b+c)(b+a)} \geq \frac{(a-b)^2}{a^2+b^2+c^2}\]

$\fbox{3}.$ In triangle $ABC$ with $AB>AC$ and incenter $I$, the incircle touches $BC,CA,AB$ at $D,E,F$ respectively. $M$ is the midpoint of $BC$, and the altitude at $A$ meets $BC$ at $H$. Ray $AI$ meets lines $DE$ and $DF$ at $K$ and $L$, respectively. Prove that the points $M,L,H,K$ are concyclic.

$\fbox{4}.$ Find all pairs of integers $(a,b)$ such that $n|( a^n + b^{n+1})$ for all positive integer $n$

Bài viết đã được chỉnh sửa nội dung bởi Phạm Quang Toàn: 23-05-2012 - 07:32

Discovery is a child’s privilege. I mean the small child, the child who is not afraid to be wrong, to look silly, to not be serious, and to act differently from everyone else. He is also not afraid that the things he is interested in are in bad taste or turn out to be different from his expectations, from what they should be, or rather he is not afraid of what they actually are. He ignores the silent and flawless consensus that is part of the air we breathe – the consensus of all the people who are, or are reputed to be, reasonable.

 

Grothendieck, Récoltes et Semailles (“Crops and Seeds”). 


#2
L Lawliet

L Lawliet

    Tiểu Linh

  • Thành viên
  • 1624 Bài viết

Day 1

$\fbox{1}.$ Given that $0 < x,y < 1$, find the maximum value of $\frac{xy(1-x-y)}{(x+y)(1-x)(1-y)}$

Day 2

$\fbox{2}.$ Let $a,b,c > 0$, prove that
\[\frac{(a-b)^2}{(c+a)(c+b)} + \frac{(b-c)^2}{(a+b)(a+c)} + \frac{(c-a)^2}{(b+c)(b+a)} \geq \frac{(a-b)^2}{a^2+b^2+c^2}\]

Khả năng có hạn nên chỉ dịch được 2 câu này thôi :(

Day 1

$\fbox{1}.$ Cho hai số $x,y$ thỏa mãn $0 < x,y < 1$, tìm GTLN của $\frac{xy(1-x-y)}{(x+y)(1-x)(1-y)}$.

Day 2

$\fbox{2}.$ Cho $a,b,c > 0$, chứng minh rằng:
\[\frac{(a-b)^2}{(c+a)(c+b)} + \frac{(b-c)^2}{(a+b)(a+c)} + \frac{(c-a)^2}{(b+c)(b+a)} \geq \frac{(a-b)^2}{a^2+b^2+c^2}\]

Thích ngủ.


#3
Tham Lang

Tham Lang

    Thượng úy

  • Thành viên
  • 1149 Bài viết
Bài 2 ta chỉ cần sử dụng CS :
$$\dfrac{(a-c)^2}{(b+c)(b+a)}+\dfrac{(c-b)^2}{(a+b)(a+c)}\ge \dfrac{(a-b)^2}{a^2+b^2+2(ab+bc+ca)}\ge \dfrac{(a-b)^2}{3a^2+3b^2+2c^2}$$

$$\dfrac{1}{(c+a)(c+b)}\ge \dfrac{1}{2c^2+a^2+b^2}$$

$$\dfrac{1}{3a^2+3b^2+2c^2}+\dfrac{1}{2c^2+a^2+b^2}\ge \dfrac{4}{4a^2+4b^2+4c^2}$$
Suy ra ĐPCM.

Bài viết đã được chỉnh sửa nội dung bởi huymit_95: 09-06-2012 - 09:46

Off vĩnh viễn ! Không ngày trở lại.......





1 người đang xem chủ đề

0 thành viên, 1 khách, 0 thành viên ẩn danh